LSAT and Law School Admissions Forum

Get expert LSAT preparation and law school admissions advice from PowerScore Test Preparation.

 Administrator
PowerScore Staff
  • PowerScore Staff
  • Posts: 8929
  • Joined: Feb 02, 2011
|
#73595
Complete Question Explanation

Must Be True. The correct answer choice is (D).

The stimulus presents a causal chain of events. First, we are told that two different factors - scant news coverage of local politics and the fact that local politics is often conducted in secret - both contribute to isolation of politicians from the electorate (voters). That isolation is then cited as a cause of reduced chances for a positive response from government. The reduced chance of such a response is said to cause lower resident participation in local politics.

The stem uses the soft "most strongly supported" language found in many Must Be True questions, but we are still looking for a claim that is based entirely on what we read in the stimulus, with no outside information or assumptions being added. Because the stimulus is causal, and because it builds a chain of causes and effects, we can take two approaches in our prephrasing: 1) The answer will be that something in the chain causes something further on down the chain (for example, conducting local politics in secret causes less chance of positive official response); or 2) removing a cause somewhere in the chain will remove or reduce an effect further on down the chain (for example, if local politics was NOT done in secret, there would be a greater chance of positive official response). These are variations of "where the cause is present, the effect is present" and "where the cause is absent, the effect is absent." Look for either of these in the answer chocies.

Answer choice (A): This is a close call, as it looks very much like removing the cause and removing the effect, but there is a problem with the answer. The effect was that a positive official response became less likely. Removing that effect would mean that a positive response would become more likely than it otherwise was, but that does not mean that it would actually be likely. The odds would be better, but not necessarily good. Maybe they would increase from 5% to 10%, still a far cry from being likely (greater than 50% chance)? A very attractive trick answer - be careful!

Answer choice (B): This answer is a loser as soon as you get to the word "should." There is no information in the stimulus about what should or should not occur, only facts about what does occur. We cannot use those facts to prove a statement that is an opinion.

Answer choice (C): We have no way of knowing what the most important factor influencing anyone is, only that some factors do have some influence. This answer is far too strong compared to the relatively weak stimulus.

Answer choice (D): This is the correct answer choice. This simply acknowledges that the relatively light coverage of local politics is contributing to the effect at the end of the causal chain, the discouragement of resident participation. Note how the answer doesn't even go so far as to say that people would be more likely to participate, making it even easier to defend based on the facts in the stimulus. It's simply "this thing is a cause, and taking it away removes at least that one cause from the equation." Exactly what we want from a Must Be True answer - something entirely based on the stimulus that is easily proven by it with no outside help.

Answer choice (E): This answer reads something like a Mistaken Reversal, a concept from Conditional Reasoning, and it is incorrect because it reverses the causal relationship. There is no reason to believe that alleviating an effect (discouragement) would have any impact on the alleged cause (isolation).
 Usma
  • Posts: 4
  • Joined: Aug 31, 2015
|
#19856
For this question I narrowed it down to D and E but chose the wrong answer. I tackled this question by creating a causal relationship.

Politics not covered thoroughly
and ---> isolation of politicians ---> discourage resident participation
business conducted in secret

I liked the sound of D because of the "at least one source of discouragement" but I thought it was a MR whereas E was the contrapositive.

Perhaps I took the wrong approach when answering this question.

Thank you.
Last edited by Usma on Fri Sep 18, 2015 8:55 pm, edited 1 time in total.
 Clay Cooper
PowerScore Staff
  • PowerScore Staff
  • Posts: 241
  • Joined: Jul 03, 2015
|
#19863
Hi Usma,

Thanks for your question. This conditional reasoning can be tricky.

D is correct because, since the stimulus tells us that both local political secrecy and insufficient news coverage contribute to politicians being isolated, which in turn contributes to a decrease in resident participation, reducing either of these original factors (local political secrecy and insufficient news coverage) would, necessarily, reduce at least one sort of discouragement against resident participation that currently exists.

E is incorrect because, though the contrapositive tells us that if resident participation were not discouraged, insufficient news coverage could not be a problem, the causality of this relationship has been lost - it is not true that increased resident participation would then have necessrily caused better news coverage.

Don't be discouraged - I think you used the correct approach, you just imperfectly applied the conditional reasoning bits of evidence.
 Usma
  • Posts: 4
  • Joined: Aug 31, 2015
|
#19868
Hi Clay,

Thank you for your explanation. I just realized that I had a typo mixing up D and E but you understood anyways.

So in other words, not discouraging resident participation has no effect on how the media covers politics.

Am I correct to say that to answer this question correctly, the effect (discourage resident participation) should be represented in the answer choice. E using said effect as the cause, does not say anything about how the news media covers politics.
 Jon Denning
PowerScore Staff
  • PowerScore Staff
  • Posts: 904
  • Joined: Apr 11, 2011
|
#19888
Hey Usma,

I see that Clay provided a great response to your first question (thanks Clay!), but figured I'd jump in and briefly address your follow up.

I don't know that "the effect" (dicouragement of resident participation) must be present in the correct answer choice, as Must Be True questions often allow for a number of potential inferences to be drawn, but given that resident participation being discouraged is sort of the "final piece" here I'd say it's very likely that the correct answer will touch on it.

And that's a typical feature of most Must questions that allow for connections/links to be drawn between concepts: usually the inference you'll draw is based to some degree off of that connection/linkage.

I hope that helps clarify things for you! Let me know if not!

Jon
 sanderina
  • Posts: 7
  • Joined: Feb 14, 2017
|
#32734
Can someone explain to me why A is not a correct answer? Thx
 Adam Tyson
PowerScore Staff
  • PowerScore Staff
  • Posts: 5191
  • Joined: Apr 14, 2011
|
#32740
Thanks for asking, sandina - answer A does look potentially attractive if you look at the chain of events here causally, because A looks a lot like "where the cause is absent, the effect is absent". Here's the problem, though - the effect here is a lower chance of a particular act eliciting a positive response, but we don't know how low that chance is and how high it might have otherwise been. Answer A says that if the cause (isolated politicians) were reduced (not eliminated - less isolation, not zero isolation), then the effect (low chance of positive response) would be completely reversed and turned into a high chance. This answer would be a lot better if it said there would be a higher chance of a positive response (more likely) if the cause were reduced, but as it is it goes too far. Maybe in the very best of cases there is still only a 30% chance of getting a positive response, but when politicians are isolated that drops down to 2%? We still could not expect a greater than 50% chance of a response after eliminating that isolation! That's what "likely" means, after all - more likely than not, or greater than 50% probability.

While answer A could be true, the stimulus doesn't give us enough to support it as a Must Be True/Most Strongly Supported answer, and answer D has so much more support for it and is therefore a better choice.

I hope that was clear! Keep on asking, we're here to help.
 sanderina
  • Posts: 7
  • Joined: Feb 14, 2017
|
#32753
Thank you Adam. Your explanation makes sense to me.
 jrc3813
  • Posts: 53
  • Joined: Apr 16, 2017
|
#34453
Clay Cooper wrote:Hi Usma,

Thanks for your question. This conditional reasoning can be tricky.

D is correct because, since the stimulus tells us that both local political secrecy and insufficient news coverage contribute to politicians being isolated, which in turn contributes to a decrease in resident participation, reducing either of these original factors (local political secrecy and insufficient news coverage) would, necessarily, reduce at least one sort of discouragement against resident participation that currently exists.

E is incorrect because, though the contrapositive tells us that if resident participation were not discouraged, insufficient news coverage could not be a problem, the causality of this relationship has been lost - it is not true that increased resident participation would then have necessrily caused better news coverage.

Don't be discouraged - I think you used the correct approach, you just imperfectly applied the conditional reasoning bits of evidence.
I didn't think you could apply conditional reasoning to this question it seemed more about causality. I thought the reason E was incorrect was because it just reversed the causality. Political isolation causes residents to be discouraged, but you can't make any causal inferences the other way. If E said "If participation was not discouraged, local politicians would not be isolated" that may be right but the fact that it said it would "cause" it seems to be the problem.

I guess the problem I'm having is seeing how conditional reasoning fits into cause and effect relationships.
 Emily Haney-Caron
PowerScore Staff
  • PowerScore Staff
  • Posts: 577
  • Joined: Jan 12, 2012
|
#34503
Hi jrc3813,

Great question! Causal reasoning and conditional reasoning aren't mutually exclusive, so you can get problems (like this one) where you have both conditional reasoning giving you a chain of conditions but also those relationships are causal in nature. As a result, E is wrong BOTH for the reason you identified (great work with that!) and for the reason Clay identified. Really, I think what you and Clay are saying is pretty similar, you're just phrasing it in different ways. I hope that helps!

Get the most out of your LSAT Prep Plus subscription.

Analyze and track your performance with our Testing and Analytics Package.